Line 43: Line 43:
 
<math>\text{5. } g \left( x^{*}  \right) \leq0</math><br>  
 
<math>\text{5. } g \left( x^{*}  \right) \leq0</math><br>  
  
'''Definision: Regular point''' &nbsp; &nbsp; &nbsp;&nbsp;
+
'''Definision: Regular point''' &nbsp; &nbsp; &nbsp;&nbsp;  
  
<math>x*{*} \text{satisfy} h\left( x^{*} \right)=0, g\left( x^{*} \right)\leq0 \text{ and let } J\left(x^{*}\right)=\left{ j:g_{j}\left(x^{*}\right)=0 \right}</math><br> &nbsp; &nbsp; &nbsp;&nbsp;<math>x^{*}\text{ is regular point if } \nabla h_{i} \left( x^{*} \right),  \nabla g_{j} \left( x^{*} \right),  1\leq i\leq m, j\in J \left( x^{*} \right)</math>  
+
<font color="#ff0000"><span style="font-size: 17px;">'''&nbsp; &nbsp; &nbsp;<math>x^{*} \text{ satisfy } h\left( x^{*} \right)=0, g\left( x^{*} \right)\leq0 \text{ and let } J\left(x^{*}\right)=\left{ j:g_{j}\left(x^{*}\right)=0 \right}</math>
 +
'''</span></font> &nbsp; &nbsp; &nbsp;&nbsp;<math>x^{*}\text{ is regular point if } \nabla h_{i} \left( x^{*} \right),  \nabla g_{j} \left( x^{*} \right),  1\leq i\leq m, j\in J \left( x^{*} \right)</math>  
  
 
'''SONC: '''Suppose that&nbsp;<span class="texhtml">''x''<sup> * </sup></span>&nbsp;is regular<br> <math>\text{1. } \mu ^{*}\geq0 \text{, } Df\left ( x^{*} \right )+\lambda ^{*T}Dh\left ( x^{*} \right )+\mu ^{*T}Dg\left ( x^{*} \right )=0^{T} \text{, } \mu ^{*T}g\left ( x^{*} \right )=0</math>  
 
'''SONC: '''Suppose that&nbsp;<span class="texhtml">''x''<sup> * </sup></span>&nbsp;is regular<br> <math>\text{1. } \mu ^{*}\geq0 \text{, } Df\left ( x^{*} \right )+\lambda ^{*T}Dh\left ( x^{*} \right )+\mu ^{*T}Dg\left ( x^{*} \right )=0^{T} \text{, } \mu ^{*T}g\left ( x^{*} \right )=0</math>  

Revision as of 12:23, 29 June 2012

ECE Ph.D. Qualifying Exam in "Automatic Control" (AC)

Question 3, Part 2, August 2011

Part 1,2,3,4,5

 $ \color{blue}\text{5. } \left( \text{20 pts} \right) \text{ Consider the following optimization problem, } $

                            $ \text{optimize} \left(x_{1}-2\right)^{2}+\left(x_{2}-1\right)^{2} $

                        $ \text{subject to } x_{2}- x_{1}^{2}\geq0 $

                                                 $ 2-x_{1}-x_{2}\geq0 $

                                                 $ x_{1}\geq0. $

$ \color{blue} \text{The point } x^{*}=\begin{bmatrix} 0 & 0 \end{bmatrix}^{T} \text{ satisfies the KKT conditions.} $


Theorem:

For the problem:    minimize  $ f \left( x \right) $

                             subject to   $ h \left( x \right) =0 $

                                                $ g \left( x \right) \leq 0 $

The KKT condition (FONC) for local minimizer x *  of f is:

$ \text{1. } \mu^{*}\geq0 $

$ \text{2. } Df\left ( x^{*} \right )+\lambda ^{*T}Dh\left ( x^{*} \right )+\mu ^{*T}Dg\left ( x^{*} \right )=0^{T} $

$ \text{3. } \mu ^{*T}g\left ( x^{*} \right )=0 $

$ \text{4. } h\left ( x^{*} \right )=0 $

$ \text{5. } g \left( x^{*} \right) \leq0 $

Definision: Regular point       

     $ x^{*} \text{ satisfy } h\left( x^{*} \right)=0, g\left( x^{*} \right)\leq0 \text{ and let } J\left(x^{*}\right)=\left{ j:g_{j}\left(x^{*}\right)=0 \right} $       $ x^{*}\text{ is regular point if } \nabla h_{i} \left( x^{*} \right), \nabla g_{j} \left( x^{*} \right), 1\leq i\leq m, j\in J \left( x^{*} \right) $

SONC: Suppose that x *  is regular
$ \text{1. } \mu ^{*}\geq0 \text{, } Df\left ( x^{*} \right )+\lambda ^{*T}Dh\left ( x^{*} \right )+\mu ^{*T}Dg\left ( x^{*} \right )=0^{T} \text{, } \mu ^{*T}g\left ( x^{*} \right )=0 $

$ \text{2. For all } y\in T\left( x^{*} \right ) \text{, we have } y^{T}L\left ( x^{\ast },\mu ^{\ast }, \lambda ^{\ast }\right )y\geq 0 $

       $ T\left( x^{* } \right)= \left \{ y\in\Re^{n}: Dh\left( x^{*} \right)y=0, Dg_{j}\left( x^{*} \right)y=0, j\in \tilde{J}\left( x^{*} \right) \right \} $

SOSC: There exist a feasible point xthat 

$ \text{1. } \mu ^{*}\geq0 \text{, } Df\left ( x^{*} \right )+\lambda ^{*T}Dh\left ( x^{*} \right )+\mu ^{*T}Dg\left ( x^{*} \right )=0^{T} \text{, } \mu ^{*T}g\left ( x^{*} \right )=0 $

$ \text{2. For all } y\in \tilde{T}\left( x^{* }\mu^{*} \right) \text{, we have } y^{T}L\left ( x^{\ast },\mu ^{\ast }, \lambda ^{\ast }\right )y\geq 0 $

$ \tilde{T}\left( x^{* },\mu^{*} \right)= \left \{ y: Dh\left( x^{*} \right)y=0, Dg_{i}\left( x^{*} \right)y=0, i\in \tilde{J}\left( x^{*},\mu^{*} \right) \right \} $

$ \tilde{J}\left ( x^{\ast },\mu ^{\ast } \right )= \left \{ i:g_{i}\left ( x^{\ast } \right ) = 0,\mu_{i}^{\ast }> 0\right \} $

Process:

a. Write down the KKT condition for this probelm

b. Find all points (and KKT multipliers) satisfying the KKT condition. In each case, determine if the point is regular.

c. Find all points in part b that also satisfy the SONC.

d. Find all points in part c that also satisfy the SOSC.

e. Find all points in part c that are local minimizers.


$ \color{blue}\left( \text{i} \right) \text{Does } x^{*} \text{ satisfy the FONC for minimum or maximum? Where are the KKT multipliers?} $

$ \color{blue}\text{Solution 1:} $

        $ f\left( x \right) = \left(x_{1}-2\right)^{2}+\left(x_{2}-1\right)^{2} $

        $ g_{1}\left( x \right)=x_{1}^{2}-x_{2} $

        $ g_{2}\left( x \right)= x_{1}+x_{2}-2 $

        $ g_{3}\left( x \right)= -x_{1} $

 $ \text{ The problem is to optimize f(x), subject to } g_{1}\leq 0, g_{2}\leq 0, g_{3}\leq 0 $

$ \text{Let } l\left( \mu ,\lambda \right)=\nabla f\left(x \right)+\mu_{1} \nabla g_{1}\left( x \right)+\mu_{2} \nabla g_{2}\left( x \right)+\mu_{3} \nabla g_{3}\left( x \right) $

                      $ =\begin{pmatrix} 2x_{1}-4\\ 2x_{2}-2 \end{pmatrix} +\mu_{1} \begin{pmatrix} 2x_{1}\\ -1 \end{pmatrix}+\mu_{2}+\begin{pmatrix} 1\\ 1 \end{pmatrix}+\mu_{3}+\begin{pmatrix} -1\\ 0 \end{pmatrix} =0 $

$ \mu_{1} g_{1}\left( x \right)+\mu_{2} g_{2}\left( x \right)+\mu_{3} g_{3}\left( x \right) $

            $ = \mu_{1} \left( x_{1}^2-x_{2} \right)+\mu_{2} \left( x_{1}+x_{2}-2 \right)+\mu_{3} \left( -x_{1} \right) =0 $

$ \text{Let } x^{*}=\begin{bmatrix} 0\\ 0 \end{bmatrix} \text{, } $

$ \left\{\begin{matrix} \nabla l\left( x,\mu \right)=\begin{pmatrix} -4+\mu_{2}-\mu_{3}\\ -2-\mu_{1}-\mu_{2} \end{pmatrix}= \begin{pmatrix} 0 \\ 0\end{pmatrix} \\ -2\mu_{2}=0 \end{matrix}\right. \Rightarrow \left\{\begin{matrix} \mu_{1}=-2\\ \mu_{2}=0\\ \mu_{3}=-4 \end{matrix}\right. $

$ \text{As } \mu^{*}\leq 0, x^{*}\begin{bmatrix} 0\\0 \end{bmatrix} \text{satisfies the FONC for maximum.} $


$ \color{blue}\text{Solution 2:} $

$ \text{ Standard form: optimize} \left(x_{1}-2\right)^{2}+\left(x_{2}-1\right)^{2} $

                                  $ \text{subject to } g_{1}\left( x \right)= x_{1}^{2}-x_{2}\leq0 $

                                                           $ g_{2}\left( x \right)= x_{1}+x_{2}-2\leq0 $

                                                           $ g_{3}\left( x \right)= -x_{1}\leq0 $

$ \text{KKT condition: (1) } Dl\left( \mu ,\lambda \right)=Df\left(x \right)+\mu_{1}Dg_{1}\left( x \right)+\mu_{2}Dg_{2}\left( x \right)+\mu_{3}Dg_{3}\left( x \right) $

                                             $ =\left [ 2x_{1}-4+2\mu_{1}x_{1}+\mu_{2}-\mu_{3}, 2x_{2}-2-\mu_{1}+\mu_{2} \right ]=0 $                                            $ \left ( 2 \right ) \mu^{T}g\left ( x \right )=0 \Rightarrow \mu_{1}\left ( x_{1}^2-x_{2} \right )+\mu_{2}\left ( x_{1}+x_{2}-2 \right ) - \mu_{3}x_{1}=0 $

                                 $ \left ( 3 \right ) \mu_{1},\mu_{2},\mu_{3}\geq 0 \text{ for minimizer} $

                                        $ \mu_{1},\mu_{2},\mu_{3}\leq 0 \text{ for maximizer} $

                                        $ \text{where } \mu^{*}=\begin{bmatrix} \mu_{1}\\ \mu_{2}\\ \mu_{3} \end{bmatrix} \text{ are the KKT multiplier.} $

$ \text{For } x^{*}=\begin{bmatrix} 0\\ 0 \end{bmatrix} \text{, } $       $ \left\{\begin{matrix} \nabla l\left( x,\mu \right)=\begin{pmatrix} -4+\mu_{2}-\mu_{3}\\ -2-\mu_{1}+\mu_{2} \end{pmatrix}=\begin{pmatrix} 0\\0 \end{pmatrix}\\ -2\mu_{2}=0 \end{matrix}\right. \Rightarrow \left\{\begin{matrix} \mu_{1}=-2\\ \mu_{2}=0\\ \mu_{3}=-4 \end{matrix}\right. $

$ \therefore x^{*}=\begin{bmatrix} 0 \\ 0 \end{bmatrix} \text{ satisfy FONC for maximum} $


$ \color{blue}\left( \text{ii} \right) \text{Does } x^{*} \text{ satisfy SOSC? Carefully justify your answer.} $

$ \color{blue}\text{Solution 1:} $

$ L\left ( x^{*},\mu^{*} \right )= \nabla l \left( x^{*},\mu^{*} \right)= \begin{pmatrix} 2&0 \\ 0&2 \end{pmatrix}-2\begin{pmatrix} 2&0 \\ 0&0 \end{pmatrix} = \begin{pmatrix} -2&0 \\ 0&2 \end{pmatrix} $

'Failed to parse (lexing error): \tilde{T}\left( x^{* }\mu^{*} \right) : \left\{ \begin{matrix} y^{T}\binom{0}{-1} =0 \\ y^{T}\binom{-1}{0} =0 \end{matrix} \right. \Rightarrow \tilde{T}\left( x^{* }\mu^{*} \right)= \left \{ \binom{0}{0} \right \}'

SOSC is trivially satisfied.

$ \color{red} \text{This solution misunderstood the range of } y \text{ for SOSC condition } y^{T}L\left ( x^{\ast },\mu ^{\ast } \right )y\geq 0 $

$ \color{red} y\in \tilde{T}\left( x^{* }\mu^{*} \right) \text{, where } \tilde{T}\left( x^{* },\mu^{*} \right)= \left \{ y:Dg_{i}\left( x^{*} \right)y=0, i\in \tilde{J}\left( x^{*},\mu^{*} \right) \right \} $


$ \color{blue}\text{Solution 2:} $

$ L\left ( x_{1}\mu \right )= D^{2} l \left ( x _{1}\mu \right )= \begin{bmatrix} 2+2\mu_{1} & 0 \\ 0 & 2 \end{bmatrix} $

                   $ \text{for point } x^{*}=\begin{bmatrix} 0 \\ 0 \end{bmatrix} \text{, we get } \mu_{1}=-2 \text{ from KKT condition.} $

$ \therefore L \left ( x^{*}, \mu ^{*}\right )=\begin{bmatrix} -2 & 0 \\ 0 & 2 \end{bmatrix} $

$ \tilde{T}\left( x^{* },\mu^{*} \right)= \left \{ y:Dg_{i}\left( x^{*} \right)y=0, i\in \tilde{J}\left( x^{*},\mu^{*} \right) \right \} $

$ \tilde{J}\left ( x^{\ast },\mu ^{\ast } \right )= \left \{ i:g_{i}\left ( x^{\ast } \right ) = 0,\mu_{i}^{\ast }> 0\right \} $     $ \therefore i= 2 $

$ \therefore \tilde{T}\left ( x^{\ast },\mu ^{\ast } \right )= \left \{ y:\left [ 1,1 \right ]y= 0 \right \}= \left \{ y:y_{1}= -y_{2} \right \} $

$ \begin{bmatrix} y_{1}& y_{2} \end{bmatrix}\begin{bmatrix} -2 & 0\\ 0 & 2 \end{bmatrix} \begin{bmatrix} y_{1}\\ y_{2} \end{bmatrix} \geqslant 0 $

                    $ -2y_{1}^{2}+2y_{2}^{2}\geqslant 0\cdots \left ( 1 \right ) $

                    for y1 = y2,  (1) is always satisfied.

$ \therefore \text{For all } y\in \tilde{T}\left( x^{* },\mu^{*} \right) \text{, we have } y^{T}L\left ( x^{\ast },\mu ^{\ast } \right )y\geq 0 $

$ \therefore \text{point } x^{*} \text{satisfy the SOSC} $


$ \color{blue}\text{Relative Problem: } $

                  minimize    − x2 + (x1 − 1)2 − 2

                  $ \text{subject to } x_{1}+x_{2}\leq2 $

$ \text{KKT condition: 1. } \mu\geq0 $

                                $ \text{2. } \begin{bmatrix} 2 \left(x_{1}-1 \right) & -1 \end{bmatrix} +\mu \begin{bmatrix} 1 & 1 \end{bmatrix} = \begin{bmatrix} 0 & 0 \end{bmatrix} $

                                        $ \text{3. } \mu \left( x_{1}+x_{2}-2 \right)=0 $

$ \text{From 2, } \mu=1 \text{, and } 2\left ( x_{1}-1 \right )=-1 $

From 3, x1 + x2 = 2

$ \text{From above two equations, we obtain a candidate point for the minimizer } x^{*}=\begin{bmatrix} 1/2\\ 3/2 \end{bmatrix} $

Check for SOSC:

$ L \left ( x^{*}, \mu ^{*}\right )= F\left ( x^{*}\right )+ \mu ^{*} G\left ( x^{*}\right )=\begin{bmatrix} 2 & 0 \\ 0 & 0 \end{bmatrix} $

Because μ * > 0 ,whave

$ \tilde{T}\left( x^{* },\mu^{*} \right)= \left \{ y: \begin{bmatrix} 1 & 1 \end{bmatrix}y=0 \right \} = \left \{ y: =\begin{bmatrix} a & -a \end{bmatrix}:a\in\Re \right \} $

$ \text{Hence } y^{T}L\left ( x^{\ast },\mu ^{\ast } \right )y=\begin{bmatrix} a & -a \end{bmatrix}\begin{bmatrix} 2 & 0\\ 0 & 0 \end{bmatrix}\begin{bmatrix} a \\ -a \end{bmatrix}=2a^{2}>0 $

$ \therefore x^{*} \text{ satisfies the SOSC for strict local minimizer. } $



Automatic Control (AC)- Question 3, August 2011

Go to


Back to ECE Qualifying Exams (QE) page

Alumni Liaison

has a message for current ECE438 students.

Sean Hu, ECE PhD 2009